Difference between revisions of "2024 SSMO Team Round Problems/Problem 8"

(Created page with "==Problem== Three integers <math>0\le a\le b\le c< 229</math> satisfy the congruence <math>n^3 \equiv 1 \pmod{229}.</math> Given that <math>71^2 - 3</math> and <math>107^2 +...")
 
 
Line 4: Line 4:
  
 
==Solution==
 
==Solution==
 +
Note that <math>n^3\equiv1\pmod{229}\implies (n^3-1)\equiv0\pmod{229}.</math> Consider the complex third roots of unity <math>1</math> and <math>\frac{-1\pm i\sqrt{3}}{2}.</math> They can be defined based on <math>\frac12,\sqrt{-1},</math> and <math>\sqrt3.</math> From the given multiples of <math>229,</math> we can <math>\sqrt{3}\equiv71\pmod{229}</math> and <math>i\equiv107\pmod{229}</math> since they function like square roots. In addition, <math>229</math> is odd so <math>2^{-1}</math> exists. We have <math>i\sqrt{3}\equiv107\cdot71\equiv269\pmod{229}.</math> Thus, <math>\frac{-1+i\sqrt{3}}{2} \equiv 134\pmod{229}</math> and <math>\frac{-1-i\sqrt{3}}{2}\equiv94\pmod{229}.</math> So, <math>a = 1,b = \boxed{94},</math> and <math>c = 134.</math>
 +
 +
~SMO_Team

Latest revision as of 14:38, 10 September 2025

Problem

Three integers $0\le a\le b\le c< 229$ satisfy the congruence $n^3 \equiv 1 \pmod{229}.$ Given that $71^2 - 3$ and $107^2 + 1$ are both multiples of $229,$ find the value of $b.$

Solution

Note that $n^3\equiv1\pmod{229}\implies (n^3-1)\equiv0\pmod{229}.$ Consider the complex third roots of unity $1$ and $\frac{-1\pm i\sqrt{3}}{2}.$ They can be defined based on $\frac12,\sqrt{-1},$ and $\sqrt3.$ From the given multiples of $229,$ we can $\sqrt{3}\equiv71\pmod{229}$ and $i\equiv107\pmod{229}$ since they function like square roots. In addition, $229$ is odd so $2^{-1}$ exists. We have $i\sqrt{3}\equiv107\cdot71\equiv269\pmod{229}.$ Thus, $\frac{-1+i\sqrt{3}}{2} \equiv 134\pmod{229}$ and $\frac{-1-i\sqrt{3}}{2}\equiv94\pmod{229}.$ So, $a = 1,b = \boxed{94},$ and $c = 134.$

~SMO_Team